0
$\begingroup$

Find the unique cases when ${t}^{2} - 4$ is a perfect square say, ${n}^{2}$, with height bound $|t| \le N$ for positive integer $N \ge 1$, when $t$ is a rational where $t = p/q$ and integers $p$ an $q$ are relatively prime, $|p| \le N$ and $1 \le q \le N$. I am looking for a counting like solution of the unique cases, exact if possible or more likely an asymptotic expansion.

For a given $N$ there are $N \left({2\, N + 1}\right)$ cases to consider. The relatively prime condition reduces this by $6/{\pi}^{2}$ which is the probability that two random integer are relatively prime. I left off the next order term. For the integer case the only solutions are for $t = \pm 2$ which is covered in my other calculations.

$\endgroup$

1 Answer 1

4
$\begingroup$

From $t^2-4=s^2$ we get $$ t^2-s^2=4~~ \Longrightarrow ~~ (t+s)(t-s) = 4 $$ hence the general rational solution $(t,s)$ is, putting $2\lambda = t+s$: $$ \left( \lambda+\frac{1}{\lambda}, \lambda-\frac{1}{\lambda} \right). $$ (It is easy to check that this indeed solves your equation.) So now we need to find the height of $t = \lambda + 1/\lambda$ in terms of the height of $\lambda=:n/d$. We have $$ t = \lambda+\frac{1}{\lambda} = \frac{d^2+n^2}{dn}, $$ where the last fraction is obviously in lowest terms, at least if $n/d$ was. Hence the height of $t$ satisfies $$ H(t) \sim H(\lambda)^2 $$ asymptotically. Since there are $O(N^2)$ rational numbers $\lambda$ of height $< N$, the number of $t$ with $H(t)<N$ that satisfies $t^2-4=\square$ is $O(N)$.

$\endgroup$
4
  • $\begingroup$ This is very nice, I did not think of these simple steps. I have from building numerical tables approximately $0.4772\, N$ as the asymptotic. So I suspected that the solution was $O(N)$. All that I need is to estimate the leading coefficient plus error if possible. $\endgroup$ Jan 27, 2016 at 2:01
  • $\begingroup$ I think it should be possible to work out the leading constant using the formula for $t$ given above, but I do not have the time to do the calculation right at this moment. But from the formula $t=(d^2+n^2)/(dn)$ we get $H(\lambda)^2=\max(\left| d \right|,\left| n \right| )^2 \leq d^2+n^2 = H(t) \leq \max( 2d^2, 2n^2 ) = 2H(\lambda)^2$. Since IIRC we have $\# \{ \lambda \in \mathbb{Q} : H(\lambda) < N \} \sim 6N^2 / \pi^2$, we get for $f(N) := \# \{ t \in \mathbb{Q} : \textrm{$H(t)<N$ and $t^2-4$ is square} \}$ that $3N / \pi^2 < f(N) < 6N / \pi^2$. $\endgroup$
    – R.P.
    Jan 27, 2016 at 2:27
  • $\begingroup$ Incidentally, $0.4772$ seems close to $3/(2 \pi)$. But that could be just a coincidence. $\endgroup$
    – R.P.
    Jan 27, 2016 at 2:39
  • $\begingroup$ In fact, yes, the leading constant seems indeed to be $3/(2 \pi)$. Given $N>0$, it is easy to see that there are $\sim N \pi/2 $ pairs $(d,n)$, with $d,n$ integers and $n>0$, such that $H(t) = d^2+n^2 < N$. (The relevant pairs $(d,n)$ describe the interior of a semicircle with radius $\sqrt{N}$ in the $(d,n)$-plane.) Arguing heuristically, taking $6/\pi^2$ as the density of coprime pairs $(d,n)$, we get $\sim N \pi /2 \cdot 6/\pi^2 = 3N /\pi$ pairs $(d,n)$ that give rise to values $t$ with $H(t)<N$. Since $(d,n)$ and $(n,d)$ yield the same $t$, the count of all $t$ comes to $3N/(2 \pi)$. $\endgroup$
    – R.P.
    Jan 27, 2016 at 3:02

Your Answer

By clicking “Post Your Answer”, you agree to our terms of service and acknowledge you have read our privacy policy.

Not the answer you're looking for? Browse other questions tagged or ask your own question.